Математикадан республикалық олимпиада, 2008-2009 оқу жылы, 11 сынып


Кез келген $0 < {{a}_{1}}\le {{a}_{2}}\le \ldots \le {{a}_{n}}$ ($n\ge 3$) сандары үшін $\dfrac{a_{1}^{2}}{{{a}_{2}}}+\dfrac{a_{2}^{3}}{a_{3}^{2}}+\ldots +\dfrac{a_{n}^{n+1}}{a_{1}^{n}}\ge {{a}_{1}}+{{a}_{2}}+\ldots +{{a}_{n}}$ теңсіздігі орындалатынын дәлелдеңіз. ( Д. Елиусизов )
посмотреть в олимпиаде

Комментарий/решение:

  7 | Модератормен тексерілді
2016-11-09 11:29:23.0 #

Используя неравенство между средним арифметическим и средним геометрическим получаем:

$$ \frac{a_1^2}{a_2}+a_2 \geq 2\sqrt{\frac{a_1^2}{a_2} \cdot a_2}=2a_1$$

$$ \frac{a_2^3}{a_3^2}+a_3+a_3 \geq 3 \sqrt[ \huge3]{ \frac{a_2^3}{a_3^2} \cdot a_3 \cdot a_3}=3a_2$$

$$\cdot \cdot \cdot $$

$$\frac{{a_k^{k + 1} }}{{a_{k + 1}^k }} \ge \left( {k + 1} \right)a_k - ka_{k + 1}. $$

для любого $k \leq n$.

Естественно $a_{n+1}=a_1$.

Складывая все эти неравенства и используя упорядоченность получаем наше неравенство.

  0
2021-04-12 21:39:42.0 #

хорошее решение.